LSAT and Law School Admissions Forum

Get expert LSAT preparation and law school admissions advice from PowerScore Test Preparation.

User avatar
 Dave Killoran
PowerScore Staff
  • PowerScore Staff
  • Posts: 5850
  • Joined: Mar 25, 2011
|
#87492
Complete Question Explanation
(The complete setup for this game can be found here: lsat/viewtopic.php?f=150&t=2097)

The correct answer choice is (B)

The question stem requires you to identify a pair of variables where at least one of the two members must always volunteer. The two most obvious answers are the pairs created by the third and fourth rules—S and V, and R and L—because during the setup we identified that each was a pair where at least one member volunteered. However, none of the answers identified contains either of the two pairs. Thus, we must look further into this problem to find the correct answer.

In the third and fourth rules the circumstance that created the result requested in this question was a negative sufficient condition paired with a positive necessary condition. In the setup discussion regarding the long chain inference, we mentioned that a number of sub-inferences could be drawn regarding the relationship of L and other variables. One such relationship is that of L and M:

G2-Q12-d1.png

If L does not volunteer, then M must volunteer, and via the contrapositive if M does not volunteer then L must volunteer. Thus, at least one of L and M must always volunteer. Accordingly, answer choice (B) is correct.

Note that answer choice (C) is incorrect because the relationship between L and V is as follows:

G2-Q12-d2.png
Thus, neither volunteers or both volunteer.
 Basia W
  • Posts: 108
  • Joined: Jun 19, 2014
|
#16273
Hello,

I was just wondering whether you had any suggested method of attacking these sorts of questions- do you look for sufficient conditions and rules such as the 3rd and 4th rule which stipulate that at least one of each variable within the pair must be in?

I chose answer choice B because I saw Leah within it and Masatoma who is sufficient for T.

Thank you for your time,

best,

Basia
 Basia W
  • Posts: 108
  • Joined: Jun 19, 2014
|
#16274
Because this is a undefined grouping game my diagram was simply limited to writing out the rules- would you have done this as well?
 David Boyle
PowerScore Staff
  • PowerScore Staff
  • Posts: 836
  • Joined: Jun 07, 2013
|
#16341
Basia W wrote:Hello,

I was just wondering whether you had any suggested method of attacking these sorts of questions- do you look for sufficient conditions and rules such as the 3rd and 4th rule which stipulate that at least one of each variable within the pair must be in?

I chose answer choice B because I saw Leah within it and Masatoma who is sufficient for T.

Thank you for your time,

best,

Basia
Hello Basia,

Yes, you look for all those things you mention above. Look for what you would in any grouping game, including people who "hate each other" (i.e., have a double not-arrow between them).
One reason to choose answer choice B is that if you put rule 1 and 4 together and do contrapositives, there's an "either/or" relationship between l and m, so that there must be at least one of them present.
As for "Because this is a undefined grouping game my diagram was simply limited to writing out the rules- would you have done this as well?", make sure all the rules connect together as much as possible, which can be considered an "inference" of sorts, and draw any inferences you can or templates that you reasonably can. Just drawing out the separate rules and failing to make any further conclusions may not be very helpful.

Hope this helps,
David
 bobbiejean
  • Posts: 6
  • Joined: May 28, 2016
|
#25788
I have read all the previous texts and not sure my question for #12 has been addressed.

I'm having a hard time understanding the difference between Game 2 inferences particularly, with Q 12 and Game 4 and the inferences. They look the same to me yet are used differently. this is the PT 58, Sept. 2009.

so with q 12 I used the relationship of F and T and chose answer A. Because I wrote that
if T --then no F nor no V.
contra positive:
if F then no T
and for some reason this is not correct?
but then you go to game 4 and want to set up similar arrangements :
if H -- then no S nor no M but here it is OK to use the contra positive
if S then no H.

clearly i am missing something and i would love to know what it is. :)
 Nikki Siclunov
PowerScore Staff
  • PowerScore Staff
  • Posts: 1362
  • Joined: Aug 02, 2011
|
#25879
Hi bobbiejean,

Thanks for your question. Games 2 and 4 on the September 2009 test are similar, yet quite different. In Game 2, the rules allow us to establish a conditional chain between the variables. In Game 4, this is a bit tricky, as Dave points out in his explanation. Check it out.

As far as Question 12 is concerned, we have the following global diagram to work with:
no L :arrow: R :arrow: M :arrow: T :arrow: no F

from T, you need to create another branch (which is difficult to represent here due to the limitations of the forum:

T :arrow: no V :arrow: S
Question 12 is asking us to identify a pair of people, at least one of whom must always be selected.

Answer choice (A): F and T. Do we need either F or T? Not really. T and F cannot both be selected, but it's entirely possible that neither is selected.

Answer choice (B): L and M. Do we need at least one of them? YES! Without L, we must have R, and therefore M (no L :arrow: R :arrow: M). By the contrapositive, without M, we cannot have R, and must therefore have L (no M :arrow: no R :arrow: L). Either way, at least one of L or M has to be selected.

Answer choice (C): L and V. Do we need at least one of L or V? Nope. If we don't have L, we must have R, M and T. But if T is selected, then V cannot be selected. Therefore, if we don't have L, we cannot have V either. Both can be absent without a problem.

Answer choice (D): R and S. It's perfectly acceptable to have neither R nor S, as long as we have V and L.

Answer choice (E): S and T. It's OK not to have either S or T. In fact, if we don't have S, we cannot have T.

Hope this clears it up!

Thanks,
 madisonzill
  • Posts: 9
  • Joined: Mar 16, 2021
|
#86574
I am having trouble understanding this question. can someone explain to me why B is the correct answer?
User avatar
 Ryan Twomey
PowerScore Staff
  • PowerScore Staff
  • Posts: 141
  • Joined: Mar 04, 2021
|
#86600
Hey Madisonzill,

Well the most important thing in this type of game is to make sure your initial conditional chain is correct. I like to connect everything into one chain in this type of game so I can see everything on the page. And then I write the contrapositive of my chain.

Then in this type of question, where they are asking which one of the following pairs of entities is it the case that at least one of them must be selected, we are looking for an answer choice where we are going from a negative in the sufficient to a positive in the necessary. If you were to connect all your conditional statements, we would have:
no L------>R-------->M which means that if you do not have L then you must have M or the contrapositive would be if you do not have M then you must have L. Either way at least one of them must be selected. So this is our correct answer.

Alternatively, if the question was, which one of the following pairs of entities is it the case that both of them cannot be selected, we would be looking for a positive in the sufficient and a negative in the necessary.

I hope this helps. First make sure your original diagram is correct and connect any conditional statements that you can.

Best,
Ryan

Get the most out of your LSAT Prep Plus subscription.

Analyze and track your performance with our Testing and Analytics Package.